PT 24, Section 3, Question 23 Forum

Prepare for the LSAT or discuss it with others in this forum.
Post Reply
msw099

New
Posts: 22
Joined: Wed Nov 11, 2009 2:19 am

PT 24, Section 3, Question 23

Post by msw099 » Tue Jan 19, 2010 4:06 am

Hi All,

Can someone please explain why answer B is correct in this stregthen question? Isn't B implying that the narrow floorboard costs less than the wide floorboard?

Thank you!

Derrex

New
Posts: 48
Joined: Tue Jul 22, 2008 1:04 am

Re: PT 24, Section 3, Question 23

Post by Derrex » Tue Jan 19, 2010 5:18 am

You need more narrow floorboards to cover the same area than if you used a wider floorboard. If they are the same price, than narrow floorboards would cost you more. Thus, it strengthens by removing the possibility that narrow floorboards are significantly cheaper.

Post Reply

Return to “LSAT Prep and Discussion Forum”